Sport/Ortho-1 Flashcards

1
Q

Stretching has NO demonstrable benefit for which one of the following?
(check one)
-Hamstring strain
-Chronic neck pain
-Joint contracture
-Osteoarthritis
-Rehabilitation post knee replacement

A

Joint Contracture

Stretching is often included in comprehensive treatment programs for musculoskeletal injuries and chronic conditions, making the determination of how much of the benefit is derived specifically from the stretching component difficult. Trials using different stretching techniques have demonstrated measurable benefit from a tailored stretching program for each of the options listed, with the exception of joint contracture.
This mobility-impairing condition results from post-healing shortness of noncontractile tissues that are not easily released with stretching.

How well did you know this?
1
Not at all
2
3
4
5
Perfectly
2
Q

A 34-year-old male presents with a 2-week history of right plantar heel pain that began after he started training for a marathon. The pain is most severe immediately upon standing in the morning and then gradually improves somewhat after ambulation. It worsens again if he stands after sitting for a period of time or after excessive walking or running.

Which one of the following is supported by evidence as a first-line intervention to provide pain relief for this condition? (check one)
Night splints
Plantar fascia stretching exercises
Acupuncture
Extracorporeal shock wave therapy
Platelet-rich plasma injection

A

Plantar fascia stretching exercises

This patient most likely has plantar fasciitis. Stretching exercises are effective in reducing heel pain caused by plantar fasciitis. Clinical trials regarding pain relief with the use of night splints are conflicting and thus inconclusive. The American College of Foot and Ankle Surgeons does not advocate for or against acupuncture to treat plantar fasciitis, as the studies available are of low quality. Extracorporeal shock wave therapy is only recommended after conservative therapies fail and for chronic plantar fasciitis. Platelet-rich plasma injections may be indicated in refractory plantar fasciitis but are not considered first-line therapy for an acute presentation.

How well did you know this?
1
Not at all
2
3
4
5
Perfectly
3
Q

A 72-year-old white male requests treatment for moderate osteoarthritis pain of the hips and knees. He has not been treated for this problem previously and has been reluctant to take medication. He takes lisinopril (Prinivil, Zestril), 20 mg daily, for hypertension, and his blood pressure is under good control. He also has a known history of stage 3 kidney disease, with a serum creatinine level of 2.1 mg/dL (N 0.6–1.5) and a glomerular filtration rate of 36 mL/min/1.73 m2. The patient’s renal function has been stable for the last 6 months. His CBC and chemistry panel are otherwise normal.

Which one of the following is the initial treatment of choice for this patient? (check one)
Acetaminophen
Celecoxib (Celebrex)
Oxycodone (OxyContin)
Sulindac (Clinoril)
Tramadol (Ultram)

A

Acetaminophen

Acetaminophen is the analgesic of choice for short-term treatment of mild to moderate pain in patients with stage 3–5 chronic kidney disease. Chronic nonterminal pain requires initial treatment with nonopioid analgesics. NSAIDs should be avoided because of the risk of nephrotoxicity.

How well did you know this?
1
Not at all
2
3
4
5
Perfectly
4
Q

A 44-year-old African-American female reports diffuse aching, especially in her upper legs and shoulders. The aching has increased, and she now has trouble going up and down stairs because of weakness. She has no visual symptoms, and a neurologic examination is normal except for proximal muscle weakness. Laboratory tests reveal elevated levels of serum creatine kinase and aldolase. Her symptoms improve significantly when she is treated with corticosteroids.

Which one of the following is the most likely diagnosis? (check one)
Duchenne’s muscular dystrophy
Myasthenia gravis
Amyotrophic lateral sclerosis
Aseptic necrosis of the femoral head
Polymyositis

A

Polymyositis

The patient described has an inflammatory myopathy of the polymyositis/dermatomyositis group. Proximal muscle involvement and elevation of serum muscle enzymes such as creatine kinase and aldolase are characteristic. Corticosteroids are the accepted treatment of choice.

It is extremely unlikely that Duchenne’s muscular dystrophy would present after age 30. In amyotrophic lateral sclerosis, an abnormal neurologic examination with findings of upper motor neuron dysfunction is characteristic. Patients with myasthenia gravis typically have optic involvement, often presenting as diplopia. The predominant symptom of aseptic necrosis of the femoral head is pain rather than proximal muscle weakness. Elevated muscle enzymes are not characteristic.

How well did you know this?
1
Not at all
2
3
4
5
Perfectly
5
Q

A 73-year-old female with diabetic neuropathy and osteoarthritis of the knees sees you to request a prescription for an assistive mobility device. The neuropathy has caused poor balance and the knee pain has made walking more painful. As a result her physical endurance has declined over the last several months.

Which one of the following assistive devices would be most appropriate for this patient? (check one)
A cane
Crutches
A walker
A wheelchair

A

A walker

A walker would be the most appropriate assistive device for this patient given her balance limitations and bilateral extremity pain. Canes are most effective for unilateral lower extremity limitations and should be held in the hand opposite the affected leg and advanced simultaneously with the affected leg. Using a cane also requires good balance and dexterity, which are limited in this patient. Crutches require significant upper body strength, balance, and increased energy expenditure, which makes their use impractical in many older adults. Wheelchairs are generally the last option, as patients who can walk should do so to maintain function and avoid deconditioning. Referral to a physical therapist can be helpful to determine the most appropriate assistive device.

How well did you know this?
1
Not at all
2
3
4
5
Perfectly
6
Q

A 30-year-old male presents to the emergency department after spraining his ankle while playing basketball. He has pain over the lateral malleolus.

Radiographs of the ankle would be indicated if he has which one of the following? (check one)
An inversion injury
Swelling over the lateral malleolus
Ecchymosis over the lateral malleolus
The inability to bear weight to walk since the injury
A previous history of ankle injury

A

The inability to bear weight to walk since the injury

The Ottawa Ankle Rules should be used to rule out fracture and prevent unnecessary radiographs. According to these guidelines, ankle radiographs are needed if there is pain over the malleolus plus bony tenderness over potential fracture areas, or an inability to bear weight and walk four steps immediately after the injury and in the emergency department or physician’s office (SOR A).

How well did you know this?
1
Not at all
2
3
4
5
Perfectly
7
Q

During a preparticipation examination of a 5-year-old male for summer soccer camp, his mother states that he frequently awakens during the night with complaints of cramping pain in both legs, and that he seems to experience this after a day of heavy physical activity. She says that she has never noticed a definite limp. A physical examination of the hips, knees, ankles, and leg musculature is entirely normal.

Which one of the following would be the most appropriate next step in the evaluation and management of this patient? (check one)
Reassurance, with no activity restrictions or treatment
Recommending that he not participate in running sports
Plain films of both hips and knees
Serum electrolyte levels
Referral to a pediatric orthopedist

A

Reassurance, with no activity restrictions or treatment

Benign nocturnal limb pains of childhood (growing pains) occur in as many as one-third of children, most often between 4 and 6 years of age. The etiology is unknown, but the course does not parallel pubescent growth, as would be expected if bone growth were the source of the pain. Pain often awakens the child within hours of falling asleep following an active day. It is generally localized around the knees, most often in the shins and calves, but also may affect the thighs and the upper extremities. A characteristic history coupled with a normal physical examination will confirm the diagnosis. Reassurance that no additional tests or treatments are necessary and that the condition is self-limiting is the most appropriate response.

How well did you know this?
1
Not at all
2
3
4
5
Perfectly
8
Q

A 34-year-old male presents with low back pain and stiffness that has been slowly worsening over the past 6 months. It is especially bothersome at night and in the morning when he gets out of bed. It improves with physical activity. He has taken ibuprofen, 400 mg several times a day, which provides moderate pain relief but is not working as well as it used to. He does not have any other joint pain, there is no history of trauma, and he is otherwise well. His BMI is 24 kg/m2. Radiographs of the lumbar spine show mild degenerative changes of the lumbar vertebrae without other abnormalities.

Which one of the following additional tests would most likely lead to a specific diagnosis? (check one)
An erythrocyte sedimentation rate
C-reactive protein
Antinuclear antibody
HLA-B27
Rheumatoid factor

A

HLA-B27

This patient’s back pain is most consistent with an inflammatory cause rather than a mechanical cause. Morning stiffness and improvement with physical activity are key features of inflammatory back pain. Ankylosing spondylitis (AS), one subset of the broader diagnostic category of axial spondyloarthritis, is the likely diagnosis in this patient. Delays in diagnosis are common due to the widespread presence of mechanical low back pain. The identification of patients with inflammatory back pain is important, because early intervention with disease-modifying agents can preserve long-term joint function. HLA-B27 is found in 74%–89% of patients with AS and it can be diagnostic in a patient with typical inflammatory back pain symptoms.

Inflammatory markers such as the erythrocyte sedimentation rate and C-reactive protein are often elevated in patients with AS but are not specific to this diagnosis. Rheumatoid arthritis is not a likely cause of back pain in this patient without any other joint findings. Antinuclear antibody testing can assist in the diagnosis of systemic lupus erythematosus, which can cause an inflammatory arthritis, but it is similarly nonspecific and lupus typically has other findings in addition to back pain.

How well did you know this?
1
Not at all
2
3
4
5
Perfectly
9
Q

A 50-year-old female with significant findings of rheumatoid arthritis presents for a preoperative evaluation for planned replacement of the metacarpophalangeal joints of her right hand under general anesthesia. She generally enjoys good health and has had ongoing medical care for her illness.

Of the following, which one would be most important for preoperative assessment of this patient’s surgical risk? (check one)
Resting pulse rate
Resting oxygen saturation
Erythrocyte sedimentation rate
Rheumatoid factor titer
Cervical spine imaging

A

Cervical spine imaging

While all of the options listed may have some value in evaluating the preoperative status of a patient with long-standing rheumatoid arthritis, imaging of the patient’s cervical spine to detect atlantoaxial subluxation would be most important for preventing a catastrophic spinal cord injury during intubation. In many cases cervical fusion must be performed before other elective procedures can be contemplated. Although rheumatoid arthritis may influence oxygen saturation and the erythrocyte sedimentation rate, these tests would not alert the surgical team to the possibility of significant operative morbidity and mortality. Resting pulse rate and a rheumatoid factor titer are unlikely to be significant factors in this preoperative scenario.

How well did you know this?
1
Not at all
2
3
4
5
Perfectly
10
Q

A 52-year-old female with a history of well-controlled diabetes mellitus presents with right shoulder pain for 2 months. She cannot recall any injury. The pain is fairly constant, has a burning quality, and disturbs her sleep.

On examination the patient has no redness or swelling. Passive and active abduction are limited to 45°. There is some limitation of shoulder flexion and internal rotation, but it is less pronounced. No focal tenderness is found. Plain films are negative.

Which one of the following is the most likely diagnosis for this patient? (check one)
Calcific tendinitis
Diabetic neuropathy
Partial rotator cuff tear
Locked posterior dislocation
Frozen shoulder

A

Frozen shoulder

Frozen shoulder is an inflammatory contracture of the shoulder capsule and mostly affects the anterosuperior and anteroinferior capsular ligaments, limiting glenohumeral movement. Diabetic patients have a 10%–20% lifetime risk of frozen shoulder. Only two other common conditions selectively limit passive external rotation: locked posterior dislocation and osteoarthritis. Plain films of the shoulder should reveal both conditions. Rotator cuff tears do not limit passive range of motion, and calcific tendinitis has a characteristic radiographic appearance.

How well did you know this?
1
Not at all
2
3
4
5
Perfectly
11
Q

A 7-year-old female is brought to your office with a complaint of right hip pain and a limp with an insidious onset. There is no history of injury or repetitive use. Her vital signs are within normal limits and she has no history of fever or chills or other systemic symptoms. On examination you note that she cannot fully abduct her hip and she winces with pain on internal rotation. A FABER test is normal. Her right leg is 2 cm (¾ in) shorter than the left. Plain films reveal flattening and sclerosis of the proximal femur with joint space widening.

What is the most likely diagnosis in this patient? (check one)
Iliopsoas bursitis
Labral tear
Legg-Calvé-Perthes disease
Septic arthritis
Stress fracture

A

Legg-Calvé-Perthes disease

Legg-Calvé-Perthes disease results from interruption of the blood supply to the still-growing femoral head. It occurs in children 2–12 years of age and presents with hip pain and an atraumatic limp. Common physical findings include leg-length discrepancies, and limited abduction and internal rotation. Radiographs reveal sclerosis of the proximal femur with joint space widening. MRI confirms osteonecrosis.

Septic arthritis also causes atraumatic anterior hip pain but occurs in the acutely ill, febrile patient. A CBC, erythrocyte sedimentation rate, C-reactive protein level, and guided hip aspiration are recommended if septic arthritis is suspected. A diagnosis of stress fracture should be considered in patients with a history of overuse and weight-bearing exercise. These patients have pain that is worse with activity, and pain on active leg raising. MRI can detect fractures not seen on plain films.

Iliopsoas bursitis presents with snapping or popping of the hip on extension from a flexed position. Labral tears present with sharp anterior hip pain at times, with radiation to the thigh or buttock. Usually patients will have mechanical symptoms such as clicking with activity. The FABER (flexion, abduction, external rotation) and FADIR (flexion, adduction, internal rotation) impingement tests are sensitive for labral tears.

How well did you know this?
1
Not at all
2
3
4
5
Perfectly
12
Q

A 36-year-old male is diagnosed with midsubstance Achilles tendinopathy. He has had symptoms for approximately 8 weeks.
For this patient, which one of the following would be the first-line treatment? (check one)
Tendon massage
Eccentric exercise
Iontophoresis
Therapeutic ultrasound
Electrical stimulation therapy

A

Eccentric exercise

For chronic midsubstance Achilles tendinopathy (symptoms lasting longer than 6 weeks), the preferred first-line treatment is an intense eccentric strengthening program of the gastrocnemius/soleus complex (SOR A). In randomized, controlled trials, eccentric strengthening programs have provided 60%–90% improvement in pain and function. Therapeutic modalities such as ultrasonography, electrical stimulation, iontophoresis, and massage and stretching have shown inconsistent results for helping patients achieve a long-term return to function. Surgical techniques are a last resort for severe or recalcitrant cases, but these techniques have not been consistently successful and carry additional risk.

To perform eccentric strengthening for Achilles tendinopathy the patient should stand on the ball of the injured foot with the calcaneal area of the foot over the edge of a stair step. The patient begins with a straight leg and the ankle in flexion. The ankle is then lowered to full dorsiflexion with the heel below the level of the step and then returned to flexion with the assistance of the uninjured leg.

How well did you know this?
1
Not at all
2
3
4
5
Perfectly
13
Q

A 36-year-old male who participates in his neighborhood basketball league visits your office with a 3-week history of heel pain. On examination he has pain over the medial plantar region of the right heel and the pain is aggravated by passive ankle dorsiflexion.

Which one of the following should you order to confirm the diagnosis? (check one)
Plain films of the foot
Ultrasonography of the foot
CT of the foot
MRI of the foot
No diagnostic imaging

A

No diagnostic imaging

The diagnosis of plantar fasciitis is based primarily on the history and physical examination. Patients may present with heel pain, and palpation of the medial plantar calcaneal region may elicit a sharp pain. Discomfort in the proximal plantar fascia can be elicited by passive ankle/first toe dorsiflexion. Diagnostic imaging is rarely needed for the initial diagnosis of plantar fasciitis. In recalcitrant plantar fasciitis plain films may be helpful for detecting bony lesions of the foot. Ultrasonography is inexpensive and may be useful for ruling out soft-tissue pathology of the heel in some patients. While MRI is expensive, it is a valuable tool for assessing causes of recalcitrant heel pain.

How well did you know this?
1
Not at all
2
3
4
5
Perfectly
14
Q

A 55-year-old male presents with severe pain, swelling, and erythema in his left first metatarsophalangeal joint. His symptoms started yesterday and he has never had this problem in the past. He has a history of hypertension, but normal renal function and no diabetes mellitus. There is no overlying skin lesion or obvious source of infection.

Which one of the following would be the most appropriate treatment for this patient? (check one)
Allopurinol (Zyloprim)
Cephalexin (Keflex)
Colchicine (Colcrys)
Febuxostat (Uloric)

A

Colchicine (Colcrys)

This patient has a classic presentation of podagra (acute metatarsophalangeal joint gout). Without an overlying skin lesion as an indicator of infection, this patient can be assumed to have gout in this classic presentation. Low-dose colchicine, 1.2 mg initially, followed by 0.6 mg in 1 hour, is recommended over high-dose colchicine, 1.2 mg initially, followed by 0.6 mg hourly for 6 hours. The high-dose regimen increases side effects but the effectiveness is not improved. This case should not be assumed to represent a septic joint and treated with cephalexin, given the typical podagra presentation. Febuxostat and allopurinol are urate-lowering drugs used as treatment for intercritical gout and not for acute treatment. Generally, treatment with urate-lowering therapy is not necessary in patients having fewer than two attacks per year.

How well did you know this?
1
Not at all
2
3
4
5
Perfectly
15
Q

Hyperbaric oxygen treatment has been shown to be beneficial for which one of the following conditions? (check one)
Tinnitus
Malignant otitis externa
Crush injury wounds
Nonunion of bone fractures
Vascular dementia

A

Crush injury wounds

Medical hyperbaric oxygen is considered a reimbursable treatment option by many insurers for a long list of diagnoses. The list of conditions shown to benefit from hyperbaric oxygen is a much shorter one, however, and includes decompression sickness and wounds caused by crush injuries. Hyperbaric oxygen treatment has been shown to improve diabetic foot ulcers in the short term but studies have so far failed to prove long-term benefit.

How well did you know this?
1
Not at all
2
3
4
5
Perfectly
16
Q

A 55-year-old female presents for a telehealth visit because of a 2-month history of right lateral hip pain. Her symptoms began shortly after she started jogging. The pain worsened a week ago after a long car trip. She reports that the pain has been interfering with her sleep quality, particularly if she rests on her right side. You ask her to walk away from her video camera so you can observe her ambulation. You note a Trendelenburg gait with her body shifting to the right side.

Which one of the following is the most likely diagnosis? (check one)
Femoroacetabular impingement
Greater trochanteric pain syndrome
A hamstring strain
A labral tear
Sacroiliac joint dysfunction

A

Greater trochanteric pain syndrome

This patient presents with lateral hip pain and symptoms of greater trochanteric pain syndrome, which can include bursitis as well as gluteus medius tendinopathy or tears. She exhibits a Trendelenburg gait, which indicates gluteus muscle weakness. Both femoroacetabular impingement and labral tears generally cause anterior hip pain. Femoroacetabular impingement is one of the most common causes of hip pain in young adults and is usually caused by a cam deformity and/or a pincer deformity of the hip joint. Labral tears are usually associated with a history of trauma or sports-related injury. Hamstring injuries and sacroiliac joint dysfunction generally cause posterior hip pain. Hamstring strains are also associated with a history of trauma, sports-related injury, or overuse

How well did you know this?
1
Not at all
2
3
4
5
Perfectly
17
Q

A 17-year-old female sees you for a preparticipation evaluation for cross country running. She has not had any falls or other injuries and estimates that she runs 40 miles per week. Menarche occurred at age 12 but over the past year her periods have become irregular, with her last menses occurring about 4 months prior to presentation. She is not sexually active and a urine pregnancy test is negative. She and her parents tell you that she eats a healthy diet, although she does report an inadvertent weight loss of 15 lb as she has increased her running mileage over the past year. She takes a multivitamin and occasional acetaminophen for pain but does not take any chronic medications.

A physical examination reveals a thin female who does not show any signs of acute distress. Her vital signs include a weight of 52 kg (115 lb), a height of 173 cm (68 in), a BMI of 18 kg/m2, a heart rate of 50 beats/min, and a blood pressure of 85/44 mm Hg.

Which one of the following is the most likely underlying cause of her condition? (check one)
Anemia
Anorexia nervosa
Low circulating estrogen levels
Low energy availability relative to needs
Vitamin D deficiency

A

Low energy availability relative to needs

This patient has the female athlete triad, a syndrome characterized by low energy availability relative to needs, disordered menses (delayed menarche, oligomenorrhea, or secondary amenorrhea), and decreased bone mineral density. This patient exhibits at least two components of the triad, although only one is required for diagnosis. Low energy availability relative to needs can be related to an eating disorder or to exercising beyond caloric supply. This leads to functional hypothalamic amenorrhea, which results in low circulating estrogen levels and then reduced bone mineral density. Anemia would be secondary to the low energy availability rather than the cause of this spectrum of issues. This patient does not have a history consistent with anorexia nervosa. Vitamin D deficiency would not cause the menstrual irregularities she has noted.

How well did you know this?
1
Not at all
2
3
4
5
Perfectly
18
Q

A 40-year-old female presents with a low back strain that occurred when moving furniture over the weekend. She rates her pain as mild to moderate. She initially tried acetaminophen, 1 g every 8 hours, and when this was ineffective, she switched to ibuprofen, 600 mg every 6–8 hours without relief. She is in good health otherwise and does not take any other medications.

Which one of the following would be the most appropriate pharmacologic therapy to recommend next? (check one)
A combination of acetaminophen, 500 mg, and ibuprofen, 600 mg, every 8 hours
CBD oil applied to the low back up to four times daily
Diclofenac topical (Voltaren Arthritis Pain) applied to the low back every 6 hours
Hydrocodone/acetaminophen (Norco), 5/325 mg every 6 hours
Oxycodone (Roxicodone), 5 mg, every 4–6 hours

A

A combination of acetaminophen, 500 mg, and ibuprofen, 600 mg, every 8 hours

This patient has acute low back pain of moderate severity. If acetaminophen and NSAIDs are ineffective when used alone, the most appropriate next step is a combination of both medications. Acetaminophen/NSAID combinations have been shown to be more effective for acute pain than either agent alone. CBD oil does not have a specific indication for acute pain, and low-quality studies show mixed results. Diclofenac topical gel is an appropriate treatment option for acute, non–low back musculoskeletal pain. This patient describes her pain as mild to moderate in severity, so other options should be tried before prescribing opioids such as hydrocodone/acetaminophen or oxycodone.

How well did you know this?
1
Not at all
2
3
4
5
Perfectly
19
Q

Current guidelines recommend moderate-intensity exercise (approximately 3.0–5.9 METs) for 150 minutes per week. Which one of the following activities is equivalent to this level of energy expenditure? (check one)
Brisk walking
High-intensity interval training
Light housework
Jogging
Sitting at a desk

A

Brisk walking

A MET is the amount of energy used by the body per minute of activity. Light intensity is <3 METs and includes activities such as sitting at a desk, light housework, casual walking, and stretching. Moderate intensity is 3.0–5.9 METs and includes brisk walking, water aerobics, and ballroom dancing. Vigorous intensity is 6 METs and is represented by activities such as high-intensity interval training, jogging, and heavy gardening.

How well did you know this?
1
Not at all
2
3
4
5
Perfectly
20
Q

Which one of the following is the best exercise to improve function in older adults living in nursing homes? (check one)
Swimming
Walking
Stretching
Stationary bicycling
Resistance training

A

Resistance training

Many types of exercise programs are beneficial for older adults, including simply walking for 30 minutes three times a week. However, a meta-analysis of progressive resistance training programs in nursing homes showed that there were significant improvements in muscle strength, chair-to-stand time, stair climbing, gait speed, and balance. This is seen even in those with advanced age, disabilities, chronic diseases, or extremely sedentary lifestyles.

How well did you know this?
1
Not at all
2
3
4
5
Perfectly
21
Q

A 40-year-old runner complains of gradually worsening pain on the lateral aspect of his foot. He runs on asphalt, and has increased his mileage from 2 miles/day to 5 miles/day over the last 2 weeks. Palpation causes pain over the lateral fifth metatarsal. The pain is also reproduced when he jumps on the affected leg. When you ask about his shoes he tells you he bought them several years ago.

Which one of the following is the most likely diagnosis? (check one)
Ligamentous sprain of the arch
Stress fracture
Plantar fasciitis
Osteoarthritis of the metatarsal joint

A

Stress fracture

Running injuries are primarily caused by overuse due to training errors. Runners should be instructed to increase their mileage gradually. A stress fracture causes localized tenderness and swelling in superficial bones, and the pain can be reproduced by having the patient jump on the affected leg. Plantar fasciitis causes burning pain in the heel and there is tenderness of the plantar fascia where it inserts onto the medial tubercle of the calcaneus.

How well did you know this?
1
Not at all
2
3
4
5
Perfectly
22
Q

A 45-year-old nurse presents with a 3-week history of heel pain that is worse at the end of a workday. She reports that there has not been any trauma. An examination is significant for tenderness inferior to the lateral calcaneus extending below the malleolus to the lateral midfoot.

Which one of the following is the most likely diagnosis? (check one)
Achilles tendinopathy
Lisfranc arthropathy
Peroneal tendinopathy
Plantar fasciitis
Tarsal tunnel syndrome

A

Peroneal tendinopathy

Peroneal tendinopathy is most commonly an overuse injury and results in tenderness along the path of the peroneal tendon from the lateral heel to the midfoot. Achilles tendinopathy involves tenderness in the posterior heel about 2–4 cm above the insertion of the Achilles tendon onto the calcaneus. Lisfranc arthropathy is caused by damage to the ligaments that support the midfoot and causes tenderness across the dorsal midfoot. Plantar fasciitis is characterized by pain and tenderness at the insertion of the plantar fascia on the plantar heel. Tarsal tunnel syndrome causes medial ankle pain that typically radiates to the medial midfoot.

How well did you know this?
1
Not at all
2
3
4
5
Perfectly
23
Q

A 4-year-old male is brought to your office by his parents who are concerned that he is increasingly “knock-kneed.” His uncle required leg braces as a child, and the parents are worried about long-term gait abnormalities. On examination, the patient’s knees touch when he stands and there is a 15° valgus angle at the knee. He walks with a stable gait.

Which one of the following should you do now? (check one)
Refer to orthopedics for therapeutic osteotomy
Refer to physical therapy for customized bracing
Prescribe quadriceps-strengthening exercises
Provide reassurance to the patient and his family

A

Provide reassurance to the patient and his family

This case is consistent with physiologic genu valgus, and the parents should be reassured. Toddlers under 2 years of age typically have a varus angle at the knee (bowlegs). This transitions to physiologic genu valgus, which gradually normalizes by around 6 years of age. As this condition is physiologic, therapies such as surgical intervention, special bracing, and exercise programs are not indicated.

How well did you know this?
1
Not at all
2
3
4
5
Perfectly
24
Q

A 55-year-old overweight male presents with a complaint of pain in the left big toe. He recently started jogging 2 miles a day to try to lose weight, but has not changed his diet and says he drinks 4 cans of beer every night. The pain has developed gradually over the last 2 weeks and is worse after running.

An examination shows a normal foot with tenderness and swelling of the medial plantar aspect of the left first metatarsophalangeal joint. Passive dorsiflexion of the toe causes pain in that area. Plantar flexion produces no discomfort, and no numbness can be appreciated.

Which one of the following is the most likely diagnosis? (check one)
Sesamoid fracture
Gout
Morton’s neuroma
Cellulitis

A

Sesamoid fracture

Pain involving the big toe is a common problem. The first metatarsophalangeal (MTP) joint has two sesamoid bones, and injuries to these bones account for 12% of big-toe injuries. Overuse, a sharp blow, and sudden dorsiflexion are the most common mechanisms of injury.

Gout often involves the first MTP joint, but the onset is sudden, with warmth, redness, and swelling, and pain on movement of the joint is common. Morton’s neuroma typically causes numbness involving the digital nerve in the area, and usually is caused by the nerve being pinched between metatarsal heads in the center of the foot. Cellulitis of the foot is common, and can result from inoculation through a subtle crack in the skin. However, there would be redness and swelling, and the process is usually more generalized
.

How well did you know this?
1
Not at all
2
3
4
5
Perfectly
25
Q

A 49-year-old white female is concerned because she has painful, cold fingertips that sometimes turn white when she is hanging out her laundry. Which one of the following medications has been shown to be useful for this patient’s condition? (check one)
Propranolol
Nifedipine (Procardia)
Ergotamine/caffeine (Cafergot)
Cilostazol (Pletal)

A

Nifedipine (Procardia)

There is no currently approved treatment for Raynaud’s disease. However, patients with this disorder reportedly experience subjective symptomatic improvement with dihydropyridine calcium channel antagonists, with nifedipine being the calcium channel blocker of choice. (1-Antagonists such as prazosin or terazosin are also effective. p-Blockers can produce arterial insufficiency of the Raynaud type, so propranolol and atenolol are contraindicated. Drugs such as ergotamine preparations can produce cold sensitivity, and should therefore be avoided in patients with Raynaud’s disease. Cilostazol is indicated for intermittent claudication but not for Raynaud’s disease.

How well did you know this?
1
Not at all
2
3
4
5
Perfectly
26
Q

A 36-year-old male laborer presents to an urgent care center 5 hours after falling off a ladder. He was 7–8 feet off the ground, and he fell directly on his anterolateral leg as he landed. Weight bearing is painful. Foot pulses are normal, as is a sensorineural examination of the foot and leg. The anterolateral lower leg is quite tender but only slightly swollen, and there is exquisite pain in that area with passive plantar flexion of the great toe. Radiographs of the lower leg and ankle are negative.

In addition to ice, elevation, and analgesia, which one of the following would be most appropriate? (check one)
Scheduled oral muscle relaxants
A 6-day oral corticosteroid taper
Physical therapy referral for early mobilization and ultrasound therapy
A short leg splint and non–weight bearing for 5–7 days
Urgent orthopedic referral for possible fasciotomy

A

Urgent orthopedic referral for possible fasciotomy

This patient most likely has acute compartment syndrome and must be urgently evaluated by an orthopedic surgeon. Typically, compartment pressure can be measured using a needle attached to a manometer, and if the pressure is elevated (usually >40 mm Hg) urgent fasciotomy is necessary to prevent muscle necrosis. If the classic “Five Ps” (pain, paresthesia, pallor, pulselessness, and paralysis) are all present, the outcome will most certainly be bad, even limb-threatening. Early identification with a high index of suspicion and urgent referral for fasciotomy is necessary to prevent tragic results.

Before the classic findings develop, patients will have tenderness out of proportion to the physical appearance of the injury and, most importantly, severe pain in the involved compartment with passive stretching of the involved muscles.

While rest, immobilization, non–weight bearing, and analgesia are all appropriate measures, none of these is sufficient treatment for this urgent problem.

How well did you know this?
1
Not at all
2
3
4
5
Perfectly
27
Q

A 24-month-old female is brought to your office by her mother because the child will not stand on her right leg. Yesterday the patient was playing at the park and her mother did not notice any injury occur. There has been no recent illness or fever. The child was born at full term, has had no medical problems, and is up to date on vaccinations.

The patient’s vital signs are normal. A physical examination reveals a healthy-appearing child in no apparent distress. She grimaces and pulls away with palpation of the right leg over the lower tibia and she will not bear weight. She has full passive range of motion of her hip, knee, and ankle joints bilaterally without apparent pain. Anteroposterior and lateral radiographs of her right tibia and fibula show no abnormalities.

Which one of the following would be the most appropriate next step in management? (check one)
Reassurance only
A CBC and C-reactive protein level
Immobilization with a cam boot, and repeat radiographs in 7 days
Bone scintigraphy
Referral to an orthopedic surgeon

A

Immobilization with a cam boot, and repeat radiographs in 7 days

A nondisplaced spiral fracture of the distal tibial shaft (toddler’s fracture) should be suspected in children from 9 months to 3 years of age who present with pain in the distal third of the tibia after minor or even unnoticed injury. Toddler’s fractures can have subtle radiographic findings and may not be visible on initial radiographs, so repeat radiography to look for healing is appropriate. Standard treatment is immobilization of the affected leg. While the fracture may heal without immobilization, reassurance alone is not recommended given the unclear diagnosis. If repeat radiography is negative and symptoms have resolved, reassurance may then be appropriate. For children with possible septic arthritis, laboratory studies should be considered, but in this case there are no signs of infection. Bone scintigraphy is more sensitive than radiography and can be considered if follow-up radiography is negative and symptoms persist. Toddler’s fractures routinely heal without complication, so referral to an orthopedic surgeon at this time would be premature.

How well did you know this?
1
Not at all
2
3
4
5
Perfectly
28
Q

A 45-year-old male works at a warehouse where he routinely lifts loads weighing more than 25 kg (55 lb). Which one of the following preventive measures has been shown to reduce the likelihood of developing chronic lower back pain? (check one)
Back braces
Over-the-counter insoles
Customized orthoses
Education on lifting techniques
Core strengthening exercises

A

Core strengthening exercises

There is reliable and consistent evidence to support core strengthening exercises as a preventive measure for low back pain. In contrast, back braces, over-the-counter insoles, customized orthoses, and education on lifting techniques have shown little or no benefit.

How well did you know this?
1
Not at all
2
3
4
5
Perfectly
29
Q

To prevent joint damage from gout, uric acid levels should be lowered by medication to (check one)
<6.0 mg/dL
<8.0 mg/dL
<10.0 mg/dL
a level that keeps the patient symptom-free for 6 months

A

<6.0 mg/dL

Targets for uric acid levels in patients with gout vary according to published guidelines but range from 5 to 6 mg/dL. Patients may be symptom-free at higher levels but risk joint damage even without acute episodes (SOR A).

How well did you know this?
1
Not at all
2
3
4
5
Perfectly
30
Q

A 54-year-old female concert pianist presents to your office with a 9-month history of searing
pain and bilateral paresthesias in the distribution of her median nerve. She says that the pain
frequently radiates as far as her shoulder, and that her fingers feel swollen even though they look
normal. She states that she has worsening paresthesias at night and often finds herself flicking
her wrist in an attempt to alleviate her symptoms.
The patient’s symptoms are reproducible with wrist flexion and she exhibits mild weakness of
the abductor pollicis brevis on examination. She has been wearing neutral wrist splints at night
for the last 8 weeks and has also been taking oral NSAIDs, resulting in only minimal relief. She
is in the middle of her concert season and is unable to take time off for a surgical procedure.

Which one of the following therapies will provide this patient with the longest symptom relief? (check one)
Full rest for 8 weeks
Full-time cock-up slinging for 8 weeks
Physical therapy
Oral corticosteroids
Local corticosteoid injection

A

Local corticosteoid injection

This patient has carpal tunnel syndrome. Initial conservative approaches for mild to moderate symptom
relief include full-time splinting for 8 weeks (SOR B) and oral corticosteroids. However, studies suggest
that local corticosteroid injections offer symptom relief for 1 month longer than oral corticosteroid therapy
and some individuals experience relief for up to 1 year. Severe or chronic symptoms usually require
surgical intervention for nerve decompression. Physical therapy is not recommended, and full rest is
unlikely in a person in a high-risk occupation for overuse syndromes.

How well did you know this?
1
Not at all
2
3
4
5
Perfectly
31
Q

An 18-year-old female presents with a painful right ankle after twisting it during a basketball game. On examination she has no tenderness over the lateral malleolus or posterior distal fibula, which she has identified as the location of the pain.

According to the Ottawa ankle rules, which one of the following would indicate that an ankle radiograph should be performed? (check one)
Moderate ankle swelling
The inability to bear weight on the right foot with the left foot elevated for 5 seconds
The inability to stand for 5 seconds with weight evenly distributed on both feet, at the time of injury and during the evaluation
The inability to take 4 steps at the time of the injury and during the evaluation
The inability to walk 6 feet during the evaluation

A

The inability to take 4 steps at the time of the injury and during the evaluation

The Ottawa ankle rules are 99% sensitive and 58% specific for identifying a fracture. They state that ankle radiography should be performed when a patient presents with pain in the malleolar region and has either point tenderness over the tip of the malleolus or the posterior edge of the affected bone (distal 6 cm), or is unable to bear weight at the time of injury and while being evaluated in the emergency department or office. Inability to bear weight is defined as the inability to take four steps. A limp when weight is transferred to the affected extremity still counts as being able to bear weight.

How well did you know this?
1
Not at all
2
3
4
5
Perfectly
32
Q

A 9-year-old male is brought to your office because he has developed a limp and refuses to bear weight on his right leg. On examination he has a temperature of 38.6°C (101.5°F) and pain with range of motion of the right hip. His WBC count and erythrocyte sedimentation rate are both elevated. A radiograph of the right hip is normal.

Which one of the following would be most appropriate at this point? (check one)
A repeat radiograph in 48 hours
Ultrasonography of the right hip
CT of the right hip
MRI of the right hip
A bone scan of the lumbar spine, right hip, and right femur

A

Ultrasonography of the right hip

The most likely diagnosis is septic arthritis of the hip. Ultrasonography is highly sensitive for the effusion seen in septic arthritis, which can be aspirated to confirm the diagnosis (SOR A). It is important to diagnose this problem as soon as possible. Clinical features of septic arthritis include an oral temperature >38.5°C (101.3°F), refusal to bear weight on the affected leg, an erythrocyte sedimentation rate >40 mm/hr, a peripheral WBC count >12,000/mm3, and a C-reactive protein level >20 mg/L. If ultrasonography is negative, a bone scan should be done. CT of the hip is indicated to visualize cortical bone. MRI is especially valuable for osteomyelitis.

How well did you know this?
1
Not at all
2
3
4
5
Perfectly
33
Q

A 45-year-old female presents to your office with knee pain. She was playing volleyball yesterday when she collided with another player and was unable to continue playing because of pain in her knee. The knee was swollen this morning. She is able to walk but not without pain, and she also has pain when she attempts to bend her knee. On examination there is medial joint line tenderness and a positive Thessaly test.

Which one of the following is the most likely cause of her knee pain? (check one)
Osteoarthritis
Anterior cruciate ligament tear
Collateral ligament tear
Medial meniscus tear
Tibial plateau fracture

A

Medial meniscus tear

A medial meniscus tear is the most likely diagnosis in a patient older than 40 who was bearing weight when the injury occurred, was unable to continue the activity, and has a positive Thessaly test. This test is performed by having the patient stand on one leg and flex the knee to 20°, then internally and externally rotate the knee. The presence of swelling immediately after the injury makes an internal derangement of the knee more likely, so osteoarthritis is less probable. This patient is able to bear weight, so a fracture is also not likely. Either a collateral ligament tear or an anterior cruciate ligament tear is possible, but these are not as common in this situation.

How well did you know this?
1
Not at all
2
3
4
5
Perfectly
34
Q

Long-term alleviation of carpal tunnel syndrome in patients with persistent symptoms is best accomplished by which one of the following? (check one)
Splinting
Physical therapy
Ibuprofen
Corticosteroid injection
Surgery

A

Surgery

NSAIDs, pyridoxine, and diuretics have been shown to be no more effective than placebo in the treatment of patients with carpal tunnel syndrome. Splinting, physical therapy, and corticosteroid injections have all been shown to result in short-term improvement. Patients with persistent symptoms achieve the best long-term relief with surgery.

35
Q

An 11-year-old female who plays in a local youth soccer league presents with right heel pain that has persisted for several months. She does not recall a specific injury that could have caused the pain. On examination the skin is intact and she is tender over the right posterior heel. Examination of the ankle and forefoot is unremarkable and a neurovascular examination is normal. You decide that rest and physical therapy would be the best initial management.

Which one of the following is CONTRAINDICATED for this patient? (check one)
Ice packs
Moist heat
Whirlpool therapy
An exercise prescription
Therapeutic ultrasound

A

Therapeutic ultrasound

Calcaneal apophysitis, also called Sever’s disease, is a common cause of heel pain in young athletes, especially those who participate in basketball, soccer, track, and other sports that involve running. Typically the heel apophysis closes by age 15. Treatment options include activity modification, the use of ice packs and/or moist heat, stretching, analgesics, and orthotic devices. The use of therapeutic ultrasound on the active bone growth plates in children is contraindicated.

36
Q

A 17-year-old female presents to your office with anterior knee pain. She tells you she recently started a running program. She says the pain is worse running down hills, and is vaguely localized just medial to the patella. Examination of the knee shows no effusion or instability, and there is no joint-line pain or patellar tenderness. McMurray’s maneuver is negative. Plain radiographs of the knee appear normal.

Which one of the following would be most appropriate at this point?

(check one)
MRI of the knee
Modification of her running program and a quadriceps and hip strengthening program
Static stretching of the quadriceps and hamstrings prior to running
A corticosteroid injection in the area of the pes anserine bursa

A

Modification of her running program and a quadriceps and hip strengthening program

This patient is suffering from patellofemoral pain syndrome, which causes anterior knee pain that is worse with running downhill. The examination is often normal, although there may be apprehension when the knee is extended with pressure over the patella and the patella will sometimes track laterally. Patellofemoral pain syndrome can be treated with exercises to strengthen the quadriceps and hips, and by using a knee sleeve with a doughnut-type cushion that the patella fits into. Static stretching would not address the problem. MRI would be indicated if there were joint-line pain or an unstable knee. Pes anserine bursitis usually causes pain and tenderness medially, below the joint line.

37
Q

A 50-year-old female reports a 1-month history of pain in her wrists. She does not recall any injury. On examination both wrists are warm but not red, feel boggy on palpation, and lack 30° of both flexion and extension. No other joints are affected. She feels fatigued and unwell, but attributes this to her busy schedule. Radiographs of the wrists are normal. Laboratory findings are unremarkable except for a mildly elevated erythrocyte sedimentation rate and a negative rheumatoid factor.

Which one of the following is the most likely diagnosis?
(check one)
Rheumatoid arthritis
Osteoarthritis
Inapparent injury
Fibromyalgia
Lyme disease

A

Rheumatoid arthritis

Rheumatoid arthritis is most often symmetric at presentation and particularly affects the wrists and other extremity joints that have a high ratio of synovium to articular cartilage. Rheumatoid factor is often negative in the early months of the disease, although it may be positive later. Radiographs and laboratory tests are helpful, but the diagnosis is primarily clinical. Osteoarthritis of the wrists usually involves the carpal-metacarpal joint of the thumb primarily, and the joint would be red if there were an injury. Fibromyalgia usually involves the soft tissue of the trunk, and there is no evidence of inflammation. Lyme disease can cause a variety of joint diseases, but not chronic symmetric arthritis.

38
Q

A 35-year-old white female complains of severe pain in her right shoulder. She notes that at night the pain intensifies if she rolls onto her right side. She has marked pain with no weakness on abduction of the shoulder; range of motion of the shoulder is normal.

The most likely cause of her pain is?
(check one)
Bicipital tendinitis
Complete rotator cuff tear
Osteoarthritis of the shoulder
Subacromial bursitis
Frozen shoulder

A

Subacromial bursitis

Acute subacromial bursitis is common and is often associated with calcific deposits in the supraspinatus tendon, pain on abduction, and local tenderness. Bicipital tendinitis results in tenderness on palpation of the tendon of the long head of the biceps. A rotator cuff tear usually results from an injury, and affects range of motion. Osteoarthritis seldom causes acute, severe pain. A frozen shoulder may result from subacromial bursitis and presents with limitation of shoulder motion.

39
Q

Static stretching before running has been shown to?

(check one)
Increase endurance
Decrease the frequency of lower limb muscle injury
Reduce delayed-onset muscle soreness
Have no benefit

A

Have no benefit

Once considered generally beneficial to the running athlete, preparticipation static stretching has been found lacking in terms of benefit and even detrimental when subjected to scientific study. There is strong evidence that static stretching significantly slows performance in sprints up to 100 meters. Studies have failed to demonstrate that static stretching before running significantly decreases the likelihood of muscular injury of the lower limbs or results in a measurable reduction of delayed-onset muscle soreness. Limited evidence suggests that preparticipation static stretching, when performed alone, adversely affects both strength and endurance in elite athletes but has little measurable effect on amateur and casual athletes. Based on current understanding of sports performance, static stretching is of most benefit when performed during the cool-down period following exercise, which has been found to increase flexibility, and is best avoided immediately before athletic endeavors. A preparatory aerobic warm-up combined with dynamic range-of-motion exercises may be of some benefit for runners.

40
Q

A 17-year-old football player presents to your office on a Monday with a right knee injury. He injured the knee in Friday night’s game when an opposing player fell against the knee from the front while the patient had his right foot planted. He was unable to bear weight after the injury, and noted immediate swelling of the knee.

A positive result with which one of the following would indicate an anterior cruciate ligament tear?
(check one)
Ballottement test
Lachman test
McMurray test
Posterior drawer test
Thessaly test

A

Lachman test

A positive Lachman test indicates that the anterior cruciate ligament may be torn. The posterior drawer test evaluates posterior cruciate ligament stability. The McMurray and Thessaly assessments test for meniscal tears. The ballottement test is for detecting intra-articular knee effusion.

41
Q

A 20-year-old female distance runner presents with a 1-month history of left knee pain. The pain is worse with her first few steps in the morning and when going down stairs. Examination of the knee reveals no deformity, effusion, or ligamentous laxity. The knee joint and surrounding tissues are not tender to palpation, with the exception of an area 2 cm proximal to the left lateral joint line.
What is the most likely cause of this patient’s pain?
(check one)
Osteoarthritis of the knee joint
Pes anserine bursitis
Iliotibial band syndrome
Chronic lateral meniscal tear
Osgood-Schlatter disease

A

Iliotibial band syndrome

This case illustrates the classic history and physical findings of iliotibial band syndrome. Pain occurs most frequently at the site where the tendon crosses over the lateral femoral epicondyle. With osteoarthritis or a meniscal tear there would be pain in the joint space with palpation. Osgood-Schlatter disease is more common in younger adolescents and is characterized by tenderness of the tibial tubercle at the distal insertion of the patellar ligament. Pes anserine bursitis is characterized by pain in the medial knee distal to the joint space, at the conjoined tendon of the sartorius, gracilis, and semitendinosus.

42
Q

A 3-year-old female is brought to your office for evaluation of mild intoeing. The child’s patellae face forward, and her feet point slightly inward.

Which one of the following would be most appropriate?
(check one)
Reassurance and continued observation
Foot stretching exercises
Orthotics
Night splints
Surgery

A

Reassurance and continued observation

Intoeing, as described in this patient, is usually caused by internal tibial torsion. This problem is believed to be caused by sleeping in the prone position and sitting on the feet. In 90% of cases internal tibial torsion gradually resolves without intervention by the age of 8. Avoiding sleeping in a prone position enhances resolution of the problem. Night splints, orthotics, and shoe wedges are ineffective. Surgery (osteotomy) has been associated with a high complication rate, and is therefore not recommended in mild cases before the age of 8.

43
Q

A 70-year-old retired farmer presents with an angulated right knee and a painful hip. He asks you about the possibility of having knee replacement surgery, although he is not eager to do so.

You would advise him that the major indication for knee replacement is?
(check one)
Severe joint pain
Marked joint space narrowing on radiologic studies
Destruction and loss of motion of the contralateral joint
An acutely infected joint

A

Severe joint pain

The major indication for joint replacement is severe joint pain. Loss of joint function and radiographic evidence of severe destruction of the joint may also be considered in the decision. The appearance of the joint and the status of the contralateral joint may be minor considerations. Surgical insertion of a foreign body into an infected joint is contraindicated.

44
Q

Which one of the following disorders is caused by an underlying mechanism of osteochondrosis rather than apophysitis? (check one)
Legg-Calvé-Perthes disease
Osgood-Schlatter disease
Sever’s disease
Sinding-Larsen-Johansson syndrome

A

Legg-Calvé-Perthes disease

Osteochondrosis refers to degenerative changes in the epiphyseal ossification areas of growing bones. Legg-Calvé-Perthes disease is a type of osteochondrosis that affects the femoral head. Patients with Legg-Calvé-Perthes disease should be referred to an orthopedist and instructed to avoid all weight-bearing activities until reossification occurs. Osteochondrosis should be differentiated from apophysitis because the etiologies and management strategies differ. Apophysitis is a traction injury to the cartilage and bony attachments of tendons in growing children. Osgood-Schlatter disease, Sever’s disease, and Sinding-Larsen-Johansson syndrome are apophysitis disorders that affect the anterior tibial tubercle, posterior heel, and inferior patellar pole, respectively. Treatment of apophysitis involves stretching, activity modification, icing, and limited use of NSAIDs.

45
Q

The diagnosis of Osgood-Schlatter disease (osteochondritis of the tibial tubercle apophysis) is best made on the basis of findings from:

(check one)
the history and examination
evaluation by an orthopedic specialist
radiographs
ultrasonography
MRI

A

the history and examination

Osgood-Schlatter disease is an inflammatory condition that is a common cause of knee pain in children and adolescents. The diagnosis is usually based on clinical findings, although radiographs may be necessary to rule out fractures or other problems if findings are not typical. MRI, ultrasonography, and orthopedic referral are not usually needed. The problem is typically self-limited and responds to activity modification, over-the-counter analgesics, stretching, and physical therapy.

46
Q

A 59-year-old male college professor presents with a 2-month history of right medial knee pain. There is no history of injury or overuse. He has no other specific joint pain except for occasional myalgias and arthralgias of his legs and arms. On most days he has morning stiffness lasting 15–20 minutes after getting out of bed. A review of systems is otherwise negative.

On examination the right knee has full range of motion. There is tenderness at the medial joint line, but no clicking or ligamentous instability. There is crepitus with movement in both knees.

Which one of the following diagnostic tests would be most appropriate at this time?
(check one)
Serologic testing
Synovial fluid analysis
Plain radiography
MRI without contrast
MRI with contrast

A

Plain radiography

The most likely diagnosis for this patient’s knee pain is osteoarthritis. While he is likely to have disease in both knees, it is common for patients to have unilateral symptoms, especially early on. Although osteoarthritis is mainly a clinical diagnosis, plain radiography is the diagnostic study of choice if there is concern about other diagnostic possibilities. Narrowing of the medial compartment of the knee joint is typically the first radiographic finding; osteophytes are also commonly seen on plain films. In the scenario presented here, there is no need for laboratory testing at this time.

47
Q

Which one of the following is most suggestive of plantar fasciitis?

(check one)
Heel pain at rest
A heel spur on radiographs
Prompt relief with NSAIDs
Heel pain that is worse with the first steps in the morning

A

Heel pain that is worse with the first steps in the morning

Plantar fasciitis is characterized by pain that is worse with the first few steps in the morning or after a
prolonged rest. NSAIDs may help with the discomfort, but prompt relief of the pain by any modality is not common. The pain is typically in the medial heel. While 50% of people with plantar fasciitis have heel spurs on radiographs, this finding is not causative or diagnostic. The diagnosis is made clinically.

48
Q

A 72-year-old male is brought to your office by a friend because of increasing confusion, irritability, and difficulty walking. This began shortly after the patient’s car broke down in a rural area and he had to walk a mile to get to a phone and call the friend. The temperature outdoors has been near 100°F.

On examination the patient has a rectal temperature of 39.5°C (103.1°F), a pulse rate of 110 beats/min, and a blood pressure of 100/60 mm Hg. His shirt is still damp with sweat.

Which one of this patient’s findings indicates that he has heatstroke rather than heat exhaustion?
(check one)
Confusion
Sweating
His temperature
His heart rate
His blood pressure

A

Confusion

A Heat exhaustion and heatstroke are both on the continuum of heat-related illness. Heatstroke is a much
more severe condition than heat exhaustion. Evidence of central nervous system dysfunction is evidence of heatstroke rather than heat exhaustion, even if other symptoms are not severe and point to heat exhaustion. Heatstroke is a medical emergency.

49
Q

A 17-year-old high school football player presents the Monday after a weekend game during which he attempted a tackle and caught his left ring finger on his opponent’s uniform. On examination he has mild swelling and tenderness over the volar aspect of the affected finger, and he is unable to actively flex the distal interphalangeal joint. Plain radiographs demonstrate no bony trauma.

Which one of the following management options would be most appropriate? (check one)
Early referral to occupational therapy for exercises to restore active range of motion
Corticosteroid injection into the flexor tendon sheath to restore active range of motion
Two weeks of immobilization in flexion, followed by buddy taping to the middle finger
Six weeks of full-time immobilization in an extension splint
Urgent referral to an orthopedist for surgical repair of the flexor tendon

A

Urgent referral to an orthopedist for surgical repair of the flexor tendon

This patient has sustained an avulsion of the flexor digitorum profundus (FDP) tendon, also known as jersey finger. The ring finger is most often involved, representing 75% of cases. The injury occurs due to forced hyperextension of an actively flexed digit, often in collision sports that involve tackling when a finger catches on an opponent’s clothing. Patients may hold the affected finger in extension relative to the other digits when the hand is at rest. The key physical examination finding is the inability to flex the distal interphalangeal (DIP) joint. Surgical repair is the appropriate treatment for a ruptured FDP tendon, with a worsening prognosis when treatment is delayed.

Although patients with jersey finger often require extended hand rehabilitation after surgery, a referral to occupational therapy prior to expedited surgical repair would delay the definitive treatment and likely result in a worse outcome. Corticosteroid injection into the flexor tendon sheath often benefits patients with stenosing tenosynovitis of the flexor tendon, also known as trigger finger. However, injecting a corticosteroid into a ruptured FDP tendon sheath would be inappropriate for management of this injury, not only by likely causing a delay in presentation to a hand surgeon, but also by potentially increasing the perioperative risk of infection and/or poor tissue healing caused by the local corticosteroid. Temporary immobilization in partial flexion followed by buddy taping is the treatment of choice for certain injuries to the proximal interphalangeal volar plate but not for FDP tendon injuries. Full-time immobilization in an extension splint for 6–8 weeks is the recommended treatment for an extensor tendon avulsion at the DIP joint, otherwise known as a mallet finger.

50
Q

A 36-year-old male presents to the urgent care clinic 2 days after he fell on his outstretched arm while snow skiing. He waited until his return home to seek evaluation and treatment. On presentation to your clinic he reports pain located in the deltoid region that is exacerbated when lifting the arms overhead. The pain is so severe that it awakens him at night. Examination of the shoulder demonstrates weakness with external rotation, internal rotation, and abduction. The empty can and drop arm tests are positive. Initial shoulder radiographs are negative.

Based on the presentation and examination, which one of the following would be the most appropriate next step in management? (check one)
Rest, ice, compression, elevation, and home exercises
NSAIDs and physical therapy
Corticosteroid injection
MRI

A

MRI

Weakness and pain are common symptoms of rotator cuff tears. The empty can test has a sensitivity of 70% and a specificity of 81% for supraspinatus muscle tear. The drop arm test has a low sensitivity of 21% for supraspinatus muscle tear, but a very high specificity of 96%. MRI would allow evaluation of the size and degree of tear as the next step in evaluation. In large or full thickness rotator cuff tears, immediate surgical repair is recommended in otherwise young, healthy individuals. Delayed repair of such tears can result in tendon degeneration, retraction, and compromised surgical results. Surgical repair should be performed within 6 weeks of the injury. Rest, ice, compression, elevation, and home exercises; NSAIDs and physical therapy; and corticosteroid injection would not be the most appropriate options for the next step in the management of this patient’s injury.

51
Q

A 45-year-old female presents with pain in her right hand and wrist. She is right hand–dominant and most symptoms are in the right hand and wrist. She has been awakening at night with numbness of the hand that improves after she shakes her wrist. Her work as a software engineer involves lots of typing.

On examination you note a positive Tinel’s sign at the right wrist consistent with carpal tunnel syndrome. She has an important project at work over the next several months and will not be able to take any time off.

Which one of the following treatments has the best evidence for delaying the need for definitive surgical therapy? (check one)
Night splints
Physical therapy
Therapeutic ultrasound
Oral corticosteroids
Corticosteroid injection

A

Corticosteroid injection

Carpal tunnel syndrome of mild to moderate severity can be treated nonsurgically. Patients with severe
symptoms or nerve damage seen on electromyography should be referred for surgical therapy. Nonsurgical
management options include splinting, physical therapy, therapeutic ultrasound, and corticosteroids (oral
or injection). Oral prednisone, 20 mg daily, for 10–14 days improves symptoms and function compared
with placebo, for up to 8 weeks, but oral corticosteroids are less effective than corticosteroid injections.
In a 2013 double-blind, placebo-controlled, randomized clinical trial comparing methylprednisolone
injection (40 mg and 80 mg) to saline injection, patients in the 80-mg injection group were less likely to
have surgery at 12 months.
While corticosteroid injections have the best evidence for delaying the need for surgery, night splints,
physical therapy, and therapeutic ultrasound have some evidence of benefit. Splinting was found to be
effective in a Cochrane review. It is low cost and safe and especially recommended in pregnancy-related
disease. Limited evidence supports the use of physical therapy to treat carpal tunnel syndrome. Nerve glide
exercises are simple hand and finger movements that are easy to learn, can be performed at home, and can
be combined with other treatments such as splinting. Therapeutic ultrasound also has limited evidence of
benefit. It requires an experienced therapist and requires multiple sessions, typically 5 days/week for 2–4
weeks.

52
Q

A neurologically intact 77-year-old female presents with severe low back pain following a fall 2 days ago that caused her to land on her buttocks. A radiograph of her lower spine shows a compression fracture of L3 with a loss of about 50% of the vertebral body height.

Which one of the following is most appropriate at this point?

(check one)
Referral for kyphoplasty
Referral for vertebroplasty
Back bracing
Bed rest
Calcitonin-salmon (Miacalcin)

A

Calcitonin-salmon (Miacalcin)

A number of measures for managing spinal compression fractures have been evaluated. The evidence for recommending kyphoplasty is weak, and the evidence for recommending against vertebroplasty is strong. The data on bracing is inconclusive, as is the recommendation for bed rest. Calcitonin has been shown to reduce the incidence of recurrent fractures and may be useful in the relief of pain.

53
Q

A 58-year-old female with diabetes mellitus complains of 2 years of right shoulder pain, which is worse with activity. There has been no trauma. She tells you one of her friends had a similar problem and was treated successfully with “some sort of shock wave treatments.”

Which one of the following diagnoses is most likely to be successfully treated with extracorporeal shock wave therapy?
(check one)
Calcific tendinitis
Gout
Partial rotator cuff tear
Frozen shoulder
Hooked acromion

A

Calcific tendinitis

Extracorporeal shock wave therapy is effective for calcific tendinitis of the rotator cuff. Side effects include bruising and pain. Needling and irrigation, physical therapy, and cortisone injections are sometimes used in patients with acute symptoms. Endoscopic and open surgical treatments are alternatives to extracorporeal shock wave therapy in refractory cases. Extracorporeal shock wave therapy does not have an established role in gout, rotator cuff tear, frozen shoulder, or hooked acromion.

54
Q

A 3-year-old male is brought to your office by his father for evaluation of 5 days of knee pain and fever up to 101.6°F. There was no known trauma preceding these symptoms. The pain and fever respond well to oral acetaminophen but continue to recur 4 hours after each dose.
On examination the child appears well and is afebrile. He had a dose of acetaminophen about 2 hours ago. There are no signs of upper respiratory infection. Examination of the knee reveals no redness, warmth, or swelling, and you see no other skin changes. He has full range of motion of both the knee and hip without pain. You note tenderness to firm palpation of the proximal tibia. He is able to bear weight and walk but refuses to jump due to anticipation of pain in his knee. Plain films of the knee are normal.

The next step in the evaluation of this patient should include which one of the following?
(check one)
Close monitoring at home
A CBC, a C-reactive protein level, and an erythrocyte sedimentation rate
Ultrasonography of the hip
Knee joint aspiration
MRI of the knee

A

A CBC, a C-reactive protein level, and an erythrocyte sedimentation rate

Joint pain in the presence of fever with no apparent source indicates a possible infection, malignancy, or rheumatologic condition and requires further workup. Laboratory evaluation, including a CBC, a C-reactive protein level, and an erythrocyte sedimentation rate can help assess for these conditions, even though none of the tests is sufficiently sensitive to rule out these diseases, and they are not specific to a single disease entity. Knee joint aspiration would be indicated to rule out septic arthritis in the presence of a joint effusion. If the hip were painful or had decreased range of motion, then ultrasonography could help identify a hip joint effusion, which would need to be aspirated. MRI may be needed in this patient, but it would likely require sedation and thus is more invasive. Starting with laboratory work is a good first step toward identifying the source of his pain and fever.

55
Q

A 15-year-old male is seen in the office for ankle pain. While playing basketball he jumped and landed on the lateral edge of his foot. He had immediate pain and did not continue playing, but was able to walk after the injury. On examination his right ankle has tenderness, swelling, and bruising over the anterior talofibular and calcaneofibular ligaments. There is no bony tenderness.

Which one of the following would be most appropriate at this point?
(check one)
Taping the ankle for future sports participation
An elastic compression wrap
A lace-up ankle support
A radiograph of the ankle
A below the knee cast

A

A lace-up ankle support

A lace-up ankle support reduces pain and recovery time after an ankle sprain (SOR B). The Ottawa Rules state that radiography is required only if there is pain in the malleolar or midfoot zone and either bony tenderness over an area of potential fracture (i.e., distal fibula or tibia, lateral or medial malleolus, base of the fifth metatarsal, or navicular bone) or an inability to bear weight immediately after the injury and when evaluated by a physician. This patient did not have those findings, and therefore would not need a radiograph (SOR A). A cast is not necessary for an ankle sprain. An elastic compression wrap alone is not as effective as a lace-up support. Taping of the ankle for future sports participation can reduce the risk of ankle sprains during sports, but would not be appropriate for an acute injury.

56
Q

Which one of the following medications used in the treatment of osteoporosis can also be used to treat the pain associated with acute and chronic vertebral compression fractures?

(check one)
Calcitonin-salmon (Miacalcin)
Raloxifene (Evista)
Risedronate (Actonel)
Teriparatide (Forteo)
Zoledronic acid (Reclast)

A

Calcitonin-salmon (Miacalcin)

While all of the medications listed can be used to treat osteoporosis, only calcitonin-salmon is useful in the management of pain associated with acute or chronic vertebral fractures. Calcitonin is an antiresorptive agent that has been shown to decrease the risk of vertebral fractures, but it is not considered a first-line treatment for osteoporosis because there are more effective agents. However, it does have modest analgesic properties that make it useful in the treatment of the pain associated with vertebral fractures.

57
Q

Which one of the following is the most appropriate initial management for patients with patellofemoral pain syndrome? (check one)
A medial unloading knee brace
Exercise therapy
Intra-articular corticosteroid injection
Confirmation of the diagnosis with MRI
Referral for knee arthroscopy

A

Exercise therapy

Patellofemoral pain syndrome (PFPS) involves pain around the patella during weight bearing on a flexed knee. PFPS is a clinical diagnosis and does not require imaging to make the diagnosis, although imaging may be indicated if other pathology is suspected. Exercise therapy is the mainstay of treatment. A medial unloading knee brace may be beneficial for symptomatic pain relief in knee osteoarthritis, but knee braces in general have poor evidence of benefit in PFPS. Intra-articular corticosteroids may be beneficial for pain relief in refractory cases but are not generally appropriate for initial treatment. Surgery may be indicated for refractory cases to correct patellar maltracking.

58
Q

An 11-year-old male is brought to your office for evaluation of bilateral posterior heel pain that has occurred for the past few months. He plays basketball and soccer several times a week and the pain begins several minutes into each of these activities. There is no pain at rest or with walking. The patient has not noticed any numbness, tingling, or weakness.

On examination you find no swelling or tenderness of the heel or Achilles tendon. Reflexes, strength, and range of motion at the ankle are intact, but he does have bilateral posterior heel pain when you passively dorsiflex the ankles.

Which one of the following is the most likely diagnosis?
(check one)
Achilles tendinopathy
Calcaneal apophysitis
Plantar fasciitis
Heel pad syndrome
Tarsal tunnel syndrome

A

Calcaneal apophysitis

Calcaneal apophysitis, also known as Sever disease, is the most common etiology of heel pain in children,
usually occurring between 5 and 11 years of age. It is thought that in these children the bones grow faster
than the muscles and tendons. A tight Achilles tendon then pulls on its insertion site at the posterior
calcaneus with repetitive running or jumping activities, causing microtrauma to the area. There may be
swelling and tenderness in this area and passive dorsiflexion may increase the pain. Radiography is usually
normal and therefore does not often aid in the diagnosis, but it may reveal a fragmented or sclerotic
calcaneal apophysis. Treatment involves decreasing pain-inducing activities, anti-inflammatory or analgesic
medication if needed, ice, stretching and strengthening of the gastrocnemius-soleus complex, and the use
of orthotic devices.

Plantar fasciitis and heel pad syndrome cause pain on the plantar surface of the heel rather than posteriorly.
Achilles tendinopathy causes tenderness to palpation of the Achilles tendon. Tarsal tunnel syndrome related
to compression of the posterior tibial nerve causes neuropathic pain and numbness in the posteromedial
ankle and heel.

59
Q

A 13-year-old African-American male is brought to your office by his mother for a limp that she has noticed for about 1 week. The patient admits to vague right-sided hip and knee pain present only with activity. He says the pain has never awakened him from sleep and is never present at rest. Neither the mother nor the patient has noticed any systemic symptoms such as fever, night sweats, weight loss, or appetite changes. There is no recent history of trauma.

On examination the patient’s weight is in the 90th percentile and his height is in the 50th percentile. He has an antalgic gait. Examination of the left hip is normal. Examination of the right hip is significant for decreased internal rotation. The right hip externally rotates involuntarily with passive flexion. There is no external deformity and no skin changes are noted. He has a negative FABER test. Palpation of the bursa and bone does not elicit pain. Examination of both knees is normal. Plain radiographs confirm your clinical impression.

Which one of the following is the most appropriate next step in the management of this patient?
(check one)
Reassurance with close follow-up
Physical therapy
Injection of the sacroiliac joint
Hospital admission for tests
Surgery

A

Surgery

The most likely diagnosis for this patient is stable slipped capital femoral epiphysis (SCFE). This is more frequent in males than in females, and is more common in African-Americans and Pacific Islanders than in whites. Although some patients present with pain, many present with a painless limp or vague pain. The average age of onset is 13.5 years for males and 12 years for females. Obesity is strongly associated with SCFE.
The lack of systemic symptoms makes osteomyelitis, abscess, or a septic joint much less likely. Malignancy is a possibility, but night pain would be more likely. Sacroiliitis is much less likely given a negative FABER test. The patient’s age makes transient synovitis or Legg-Calvé-Perthes disease less likely. Although muscle strain is a possibility, the physical examination findings of external rotation deformity and limited internal rotation are more specific for SCFE.
Once the diagnosis of SCFE is made, the patient should not bear weight and should be referred promptly for surgery to prevent complications.

60
Q

An 80-year-old female is admitted to your service at a skilled nursing facility 5 days after repair of a hip fracture. When you review her records you note that she has not received any previous treatment for osteoporosis. You are considering ordering zoledronic acid (Reclast) to reduce her risk of another fracture.

Which one of the following should be evaluated before administering zoledronic acid to this patient?
(check one)
Vitamin D levels
Liver enzyme levels
Estimated glomerular filtration rate
A CBC

A

Estimated glomerular filtration rate

Secondary prevention of fractures is an important component of care following a hip fracture. Options to consider include bisphosphonates, calcium supplementation, and vitamin D supplementation. Bisphosphonates, including zoledronic acid, can reduce rates of clinical fractures among patients who have had a hip fracture (SOR A). While long-term use of bisphosphonates may increase the risk of jaw osteonecrosis and anemia, a CBC is not necessary before initiating therapy. Contraindications to zoledronic acid include hypocalcemia and a creatinine clearance <35 mL/min or other evidence of acute renal impairment.

61
Q

A 13-year-old male presents with a 3-week history of left lower thigh and knee pain. There is
no history of a specific injury, and his past medical history is negative. He has had no fevers,
night sweats, or weight loss, and the pain does not awaken him at night. He tried out for his
school’s basketball team but had to quit because of the pain, which was worse when he tried to
run.

Which one of the following physical examination findings would be pathognomonic for slipped
capital femoral epiphysis?
(check one)
Excessive forward passive motion of the tibia with the knee flexed
Lateral displacement of the patella with active knee flexion
Limited internal rotation of the flexed hip
Reduced hip abduction with the hip flexed
An inability to extend the hip past the neutral position

A

Limited internal rotation of the flexed hip

Slipped capital femoral epiphysis (SCFE) typically occurs in young adolescents during the growth spurt,
when the femoral head is displaced posteriorly through the growth plate. Physical activity, obesity, and
male sex are predisposing factors for the development of this condition. There is pain with physical
activity, most commonly in the upper thigh anteriorly, but one-third of patients present with referred lower
thigh or knee pain, which can make accurate and timely diagnosis more difficult.
The hallmark of SCFE on examination is limited internal rotation of the hip. Specific to SCFE is the even
greater limitation of internal rotation when the hip is flexed to 90°. No other pediatric condition has this
physical finding, which makes the maneuver very useful in children with lower extremity pain. Orthopedic
consultation is advised if SCFE is suspected.
Hip extension and abduction are also limited in SCFE, but these findings are nonspecific. Displacement
of the patella is not associated with SCFE.

62
Q

A 36-year-old male with a history of ankylosing spondylitis and atrial fibrillation presents with a 3-week history of cough with hemoptysis, anorexia, night sweats, and an 11-lb weight loss. On examination he has rales in the right upper lobe, but there is no lymphadenopathy or hepatosplenomegaly. A chest radiograph shows a cavitary lesion in the right lung apex, with mediastinal hilar lymphadenopathy. His chronic disease symptoms have been well controlled with a combination of meloxicam (Mobic), adalimumab (Humira), esomeprazole (Nexium), ondansetron (Zofran), docusate sodium (Colace), and amiodarone (Cordarone).

Which one of the patient’s medications is most likely contributing to his current problem?
(check one)
Adalimumab
Amiodarone
Esomeprazole
Meloxicam
Ondansetron

A

Adalimumab

Tumor necrosis factor (TNF) inhibitors are currently approved by the U.S. Food and Drug Administration
(FDA) for the treatment of rheumatic diseases such as rheumatoid arthritis, ankylosing spondylitis,
psoriatic arthritis, and juvenile idiopathic arthritis. All drugs in this class carry an FDA black-box warning
about the potential for developing primary tuberculosis or reactivating latent tuberculosis. These drugs are
also associated with an increased risk for invasive fungal infections and opportunistic bacterial and viral
diseases. The FDA also warns of reports of lymphomas and other malignancies in children and adolescents
taking these drugs.

A PPD skin test should be performed prior to initiating TFN-inhibitor therapy. An induration of 5 mm or
greater with tuberculin skin testing should be considered a positive test result when assessing whether
treatment for latent tuberculosis is necessary prior to TFN-inhibitor use, even for patients previously
vaccinated with bacille Calmette-Guérin (BCG) (SOR B).

Ankylosing spondylitis patients may develop fibrosis of the upper lung fields with long-standing disease,
but esomeprazole, ondansetron, and meloxicam do not cause reactivation of tuberculosis. Amiodarone is
associated with a subacute cough and progressive dyspnea due to pulmonary toxicity (patchy interstitial
infiltrates).

63
Q

A 30-year-old female stepped off a curb earlier today and twisted her left ankle. She was able
to bear weight immediately following the injury and tried to continue her normal routine, but the
pain in her ankle and foot increased over the next few hours.

She comes to your office and your examination reveals swelling of the ankle and bruising of the
lateral foot. Tenderness to palpation is present over the distal aspect of the fibula and lateral
malleolus and to a lesser degree over the proximal fifth metatarsal. No bony tenderness is
present along the medial aspect of the ankle or foot.

According to the Ottawa Ankle Rules, which one of the following would be most appropriate
at this point?
(check one)
Radiographs of the ankle and foot
Radiographs of the foot only
Radiographs of the ankle only
No radiographs

A

Radiographs of the ankle and foot

The Ottawa Ankle Rules are widely accepted guidelines for appropriate evaluation of ankle and midfoot
injuries occurring in adults age 19 or older presenting for the first time in a clinical setting. The guidelines
utilize the historical and physical findings to determine which radiographic studies, if any, are indicated.
Patients who were able to bear weight immediately following their injury and who can take 4 steps
independently in a clinical setting require radiographic study only when the following criteria are met: pain
is present in the malleolar zone and bony tenderness of the posterior edge or tip of either malleolus is
elicited (ankle radiograph), or pain is present in the midfoot zone and bony tenderness of either the base
of the fifth metatarsal or the navicular region is present.

64
Q

A 70-year-old white female with osteoporosis sees you for follow-up a few days after an emergency department visit for an acute T12 vertebral compression fracture. The fracture was suspected clinically and on plain films; the diagnosis was confirmed with a bone scan. The emergency department physician prescribed oxycodone (OxyContin) and NSAIDs, but the patient is still experiencing considerable discomfort. In addition to increasing the dosage of oxycodone, which one of the following interventions would you suggest now to reduce the patient’s pain? (check one)
Calcitonin (Miacalcin)
Raloxifene (Evista)
Alendronate (Fosamax)
Physical therapy, including dexamethasone iontophoresis
Vertebroplasty

A

Calcitonin (Miacalcin)

Calcitonin, either intranasal or subcutaneous, provides pain relief within a few days in many patients with osteoporotic vertebral compression fractures. The remainder of the choices do not provide acute relief. Vertebroplasty/kyphoplasty procedures are generally reserved for cases in which medical management has failed.

65
Q

A 2-year-old white female is brought to your office by her parents, who are concerned about the child’s “flat feet.” On evaluation, the child’s feet are flat with weight-bearing, but with toe standing and with sitting the arch appears. You would: (check one)
Reassure the parents
Recommend orthotics
Recommend surgery
Recommend casting
Recommend foot-stretching exercises

A

Reassure the parents

Flexible flat feet as described are not pathologic unless painful, which is uncommon. Flexibility of the flat foot is determined by appearance of an arch when the feet are not bearing weight. No treatment is indicated for painless flexible flatfoot. Spontaneous correction is usually expected within 1 year of walking.

66
Q

A 78-year-old white female presents with a 3-day history of lower thoracic back pain. She denies any antecedent fall or trauma, and first noted the pain upon arising. Her description of the pain indicates that it is severe, bilateral, and without radiation to the arms or legs. Her past medical history is positive for hypertension and controlled diabetes mellitus. Her medications include hydrochlorothiazide, enalapril (Vasotec), metformin (Glucophage), and a general multivitamin. She is a previous smoker but does not drink alcohol. She underwent menopause at age 50 and took estrogen for “a few months” for hot flashes. Physical examination reveals her to be in moderate pain with a somewhat stooped posture and mild tenderness over T12–L1. She has negative straight-leg raising and normal lower extremity sensation, strength, and reflexes. Which one of the following is true regarding this patient’s likely condition? (check one)
An MRI or nuclear medicine bone scan should be performed
Prolonged (approximately 2 weeks) bed rest will increase the chance of complete recovery
Investigation for an underlying malignancy is indicated
Subcutaneous or intranasal calcitonin (Calcimar, Miacalcin) may be very helpful for pain relief

A

Subcutaneous or intranasal calcitonin (Calcimar, Miacalcin) may be very helpful for pain relief

The patient described has a classic presentation of an osteoporotic vertebral compression fracture. The diagnosis should be confirmed with a plain radiograph. Treatment is basically symptomatic, with a period of bed rest as short as possible (to avoid complications of immobility), pain medication, and bracing. Salmon calcitonin (injectable or intranasal) is often helpful in providing pain relief. Long-term management of underlying osteoporosis may help prevent future fractures

67
Q

An 80-year-old male presents with the chief complaint of a “bone spur,” describing mid-heel pain that worsens as the day progresses. The pain is not relieved with ibuprofen. Examination reveals tenderness in the central aspect of the heel and a radiograph of the foot is unremarkable. The most likely diagnosis is: (check one)
Multiple myeloma
Fat-pad atrophy
Tarsal tunnel syndrome
S1 radiculopathy
Plantar fasciitis

A

Fat-pad atrophy

Fat-pad atrophy is a common cause of heel pain in the geriatric patient, and in contrast to plantar fasciitis, causes pain as the day progresses. Plantar fasciitis classically presents as morning pain. Tarsal tunnel syndrome causes neuropathic pain in the distribution of the posterior tibial nerve, radiating into the plantar aspect of the foot toward the toes. Lumbar radiculopathy involves pain radiating down the leg into the heel, and is usually associated with weakness of dorsiflexion of the big toe and a decreased ankle reflex. Multiple myeloma would be an extremely unusual cause of heel pain; heel pain associated with cancer more commonly presents nocturnally.

68
Q

A 35-year-old white female comes to your office with a 3-month history of the gradual onset of pain and tenderness in her wrists and hands. She also complains of 1 hour of morning stiffness. She denies rash, fever, or skin changes. On physical examination she has symmetric swelling of the proximal interphalangeal joints and metacarpophalangeal joints. Motion of these joints is painful. She has no rash or mouth ulcers. Radiographs of the hands and wrists are negative, and a chest film is unremarkable. A CBC is normal, but the erythrocyte sedimentation rate is elevated at 40 mm/hr. Latex fixation for rheumatoid factor is negative, and an antinuclear antibody (ANA) test is negative.
The most likely diagnosis in this patient is
(check one)
rheumatoid arthritis
systemic lupus erythematosus
sarcoidosis
Lyme disease

A

rheumatoid arthritis

This patient has rheumatoid arthritis (RA) by symptoms and physical findings. A positive latex fixation test for rheumatoid factor is not necessary for the diagnosis. A negative rheumatoid factor does not exclude RA, and a positive rheumatoid factor is not specific. Rheumatoid factor is found in the serum of approximately 85% of adult patients with RA; in subjects without RA, the incidence of positive rheumatoid factor is 1%–5% and increases with age.
The ANA test is positive in at least 95% of patients with systemic lupus erythematosus, but in only about 35% of patients with RA. Elevation of the erythrocyte sedimentation rate is seen in many patients with RA, and the degree of elevation roughly parallels disease activity. At a mean of 6 months after the onset of Lyme disease, 60% of patients in the United States have brief attacks of asymmetric, oligoarticular arthritis, primarily in the large joints and especially in the knee.

69
Q

A 35-year-old female presents with a 3-week history of radial-sided wrist pain without acute trauma. Her symptoms are most prominent when lifting her 4-month-old infant into his car seat.

Which one of the following physical examination findings is most consistent with the likely diagnosis? (check one)
Focal tenderness in the anatomic snuffbox
Focal tenderness over the triangular fibrocartilage complex
Pain reproduced by repeated percussion over the volar wrist
Pain reproduced by thumb passive circumduction with axial load to the first carpometacarpal joint
Pain reproduced by ulnar wrist deviation with the thumb placed inside a closed fist

A

Pain reproduced by ulnar wrist deviation with the thumb placed inside a closed fist

This patient presents with a history of an atraumatic onset of radial-sided wrist pain that is typical of de Quervain’s tenosynovitis, a common overuse injury involving the tendons of the first dorsal compartment, specifically the abductor pollicis longus and extensor pollicis brevis. This injury is most common in women 30–50 years of age and often occurs in new mothers who frequently pick up a child. Pain localizes to the radial styloid and is reproduced by Finkelstein’s test, which involves placing the thumb inside a closed fist followed by ulnar deviation of the wrist. Focal tenderness in the anatomic snuffbox would suggest injury to the scaphoid, which is in the differential diagnosis for radial-sided wrist pain, although it usually occurs through direct trauma such as a fall onto an outstretched hand. Injury to the triangular fibrocartilage complex is a common cause of ulnar-sided, rather than radial-sided, wrist pain. Repeated percussion over the volar wrist with resultant pain and/or paresthesia in a median nerve distribution describes Tinel’s sign for carpal tunnel syndrome. Thumb passive circumduction with an axial load to the first carpometacarpal (CMC) joint describes the CMC grind test, which elicits pain from CMC joint arthritis.

70
Q

A 65-year-old female presents for follow-up of a DXA scan for osteoporosis screening. Results of the scan reveal osteopenia, with a T-score of –2.0.

Treatment for osteopenia is indicated when the 10-year risk of a major fracture reaches (check one)
5%
10%
20%
35%

A

20%

The National Osteoporosis Foundation recommends pharmacologic treatment when a DXA scan reveals a T-score –2.5 (the cutoff for a diagnosis of osteoporosis), or when the T-score falls between –1.0 and –2.5 (the diagnosis criterion for osteopenia) and the 10-year risk of a major fracture reaches 20%. The T-score of –2.0 places this patient in the “osteopenic” range. A 10-year probability of a hip fracture 3% is also an indication for treatment.

71
Q

You see a 30-year-old male who has just fallen on an outstretched hand. He complains of wrist pain and edema. Examination reveals tenderness over the anatomic snuffbox and over the scaphoid tubercle at the proximal wrist crease with the hand in extension. Radiographs of the wrist are negative. Which one of the following would be the most appropriate action at this point? (check one)
Order a bone scan for the next day
Order high-spatial-resolution ultrasonography of the wrist
Immobilize in a cast for 6–8 weeks
Immobilize in a thumb spica splint for 1–2 weeks and then order repeat radiographs

A

Immobilize in a thumb spica splint for 1–2 weeks and then order repeat radiographs

Snuffbox tenderness and tenderness over the scaphoid tubercle are very sensitive for fracture of the scaphoid, but their specificity is only 40% and 60% respectively. Therefore, while the lack of tenderness at these sites almost rules out fracture, further imaging is needed in positive cases. Plain films are recommended as the next step. A bone scan or follow-up films after immobilization for 2 weeks should be done if the initial films are negative. Bone scans may be negative until enough time has passed for osteoblastic activity to begin. Ultrasonography is not helpful for evaluation of scaphoid fractures.

72
Q

A 67-year-old female has a bone density study that indicates a T score of –3.5. You prescribe alendronate (Fosamax) but at her next visit she says she cannot tolerate the side effects and asks about other therapies.
Which one of the following has the best evidence for prevention of both vertebral fractures and hip fractures?
(check one)
Calcitonin-salmon (Miacalcin)
Raloxifene (Evista)
Teriparatide (Forteo)
Zoledronic acid (Reclast)

A

Zoledronic acid (Reclast)

There are a number of alternatives to the bisphosphonates. Unfortunately, efficacy data is not encouraging for most of them. Intravenous zoledronic acid has been shown to reduce both hip fracture risk and vertebral fracture risk. Teriparatide reduces vertebral fracture risk but not hip fracture risk. The same is true for raloxifene and calcitonin salmon.

73
Q

A 70-year-old female had a lumbar vertebral fracture 3 years ago. At that time she had a dual-energy x-ray absorptiometry (DEXA) scan, with a T score of –2.6, and was placed on alendronate (Fosamax), calcium, and vitamin D. She recently quit smoking. Her BMI is 21. A DEXA scan today shows her bone mineral density to be –2.1. Which one of the following would be most appropriate in the management of this patient? (check one)
Replace alendronate with raloxifene (Evista)
Stop alendronate, but continue calcium and vitamin D
Add raloxifene to her regimen
Add teriparatide (Forteo) to her regimen
Make no change to her regimen

A

Make no change to her regimen

Even though the patient’s DEXA has improved and she is technically osteopenic, she still has risk factors for osteoporosis, including recent smoking, low BMI, and a prior fragility fracture. She should continue her current regimen.

74
Q

A 44-year-old female presents to your office reporting that she hurts all over. After performing a thorough history and physical examination and appropriate laboratory studies you diagnose fibromyalgia. You explain to the patient that the initial treatment recommendation with the most proven efficacy is (check one)
acupuncture
aerobic exercise
amitriptyline
duloxetine (Cymbalta)

A

aerobic exercise

Aerobic exercise, a balanced diet, good sleep hygiene, and weight reduction are appropriate strategies for the management of fibromyalgia, and treatment goals should be focused on improving function and quality of life, along with managing symptoms. According to the 2017 European League Against Rheumatism, exercise is the strongest and most critical treatment for fibromyalgia. Not only does it lessen fibromyalgia symptoms, but it can also help with coexisting conditions including sleep disorders, depression, and anxiety. While some studies show improvement in symptoms with acupuncture, most evidence is low to moderate in quality. A Cochrane review found that acupuncture was superior to no treatment at all, but not superior to sham acupuncture. Pharmacologic treatments have shown only modest benefits and are often accompanied by adverse effects, so they are best used in conjunction with nonpharmacologic therapies.

75
Q

A 35-year-old female presents with a 2-week history of right posteromedial foot and ankle pain. The pain began during a vacation that included several days of sightseeing and hiking. She does not have a history of acute injury or trauma. Her pain is worse with weight bearing and improves with relative rest and ibuprofen use.

A physical examination reveals soft-tissue swelling and tenderness along the posterior edge of the medial malleolus into the medial arch of the foot. Standing alignment, range of motion, a strength assessment, and neurovascular testing are normal. Her symptoms are unchanged by gentle, repetitive tapping over the posteromedial ankle.

Which one of the following is the most likely diagnosis? (check one)
A deltoid ligament sprain
A medial malleolar stress fracture
Peroneal tendinopathy
Posterior tibialis tendinopathy
Tarsal tunnel syndrome

A

Posterior tibialis tendinopathy

This is a typical presentation for posterior tibialis tendinopathy, a common overuse injury that presents with pain in the posteromedial foot and ankle in the distribution of the posterior tibialis tendon. If not identified and treated early, patients may develop more progressive posterior tibialis tendon dysfunction and eventual arch collapse of the foot. Deltoid ligament sprains present with pain over the medial ankle ligament complex after an acute ankle injury involving an eversion mechanism. A medial malleolar stress fracture would present with pain and bony tenderness over the medial malleolus, rather than over the course of the posterior tibialis tendon as described in this scenario. Peroneal tendon injuries may occur with overuse or an acute injury, although the pain is lateral rather than medial. Tarsal tunnel syndrome is caused by compression neuropathy due to pressure on the posterior tibial nerve and presents with pain, burning, tingling, and/or numbness in the medial midfoot and medial heel. As with posterior tibialis tendinopathy, the pain from tarsal tunnel syndrome is often worse with prolonged weight bearing. Symptoms can be reproduced by tapping over the nerve (Tinel sign), which does not occur with posterior tibialis tendinopathy. The negative Tinel sign described in this scenario is more consistent with posterior tibialis tendinopathy than tarsal tunnel syndrome.

76
Q

A 56-year-old female comes to your office because she thinks she has a herniated disc in her lower back causing sciatica. She has electric pain shooting down her left leg all the way to her toes, and she requests MRI to see if she needs surgery. She has not had any injury, saddle anesthesia, or changes in bowel or bladder habits. On examination she has 2+ symmetric deep tendon reflexes of the lower extremities, a negative bilateral straight leg raising test, and a positive log roll test. Her pain is worse with external rotation of the left hip. You suspect piriformis syndrome.

Which one of the following would be the most appropriate next step in management? (check one)
Reassurance only
MRI of her lumbar spine
Injection of the piriformis muscle with a corticosteroid and local anesthetic
Referral to a physical therapist
Referral to an orthopedic surgeon

A

Referral to a physical therapist

This patient’s history and the physical examination are classic for piriformis syndrome and she will likely recover with physical therapy and home exercises. Reassurance only is not the best management option for piriformis syndrome. An MRI may be warranted later, but it is not the first step in the workup and treatment in this patient’s case. Injecting the piriformis muscle with a corticosteroid and local anesthetic is inappropriate therapy for this condition. This patient does not have any red-flag signs for lumbar radiculopathy and therefore does not require referral to an orthopedic surgeon at this time.

77
Q

A 63-year-old female presents with pain in her right hand. She states that for 3 months she has experienced a catching sensation and her ring finger occasionally locks into a claw-like form. On examination a nodule is palpated and her pain is localized to the volar surface of the metacarpophalangeal joint on her ring finger. When instructed to clench her fist and subsequently release, her ring finger remains in a flexed position.

Which one of the following is considered first-line treatment for this condition? (check one)
Buddy taping
Transcutaneous electrical nerve stimulation (TENS) unit therapy
Corticosteroid injection
Physical therapy
Surgical release

A

Corticosteroid injection

First-line therapy for stenosing flexor tenosynovitis, or trigger finger, is a corticosteroid injection into the affected flexor tendon sheath (SOR A). It may take 1–4 weeks for the patient to experience relief. Splinting may be considered if the patient is not a candidate for corticosteroid injections. Buddy taping, transcutaneous electrical nerve stimulation (TENS) unit therapy, and physical therapy are not effective treatments for trigger finger. Surgical correction may be considered after 2–3 corticosteroid injections are attempted without improvement of symptoms.

78
Q

A 70-year-old male presents with an acutely painful, swollen right knee that developed over 2–3 days without any known cause. There is no other joint pain. He has had similar, less severe episodes off and on in the past. He feels well otherwise and has not had any fever, chills, or rash.

Examination of other joints is negative except for some degenerative changes of the distal interphalangeal joints of the fingers. Examination of the right knee is notable for warmth, redness, diffuse tenderness, and swelling. There is no evidence of knee instability, meniscal injury, or trauma. A knee x-ray shows soft-tissue swelling and chondrocalcinosis (calcification of the cartilage). Knee aspiration is performed and a synovial fluid Gram stain is negative for bacteria with other results pending.

Which one of the following is the most likely explanation for these knee findings? (check one)
Calcium pyrophosphate deposition disease (pseudogout)
Gout
Osteoarthritis
Psoriatic arthritis
Septic arthritis

A

Calcium pyrophosphate deposition disease (pseudogout)

Rheumatoid arthritis is primarily a clinical diagnosis and no single laboratory test is considered definitively diagnostic. Anti-cyclic citrullinated peptide (anti-CCP) antibody is recommended by rheumatologists to improve the specificity of testing for rheumatoid arthritis. Anti-CCP is more specific than rheumatoid factor, and may predict erosive disease more accurately. Antinuclear antibody has limited usefulness for the diagnosis of rheumatoid arthritis. Anti-Sm antibody is useful to help diagnose systemic lupus erythematosus. Nonspecific changes in complement levels are seen in many rheumatologic disorders. The erythrocyte sedimentation rate is useful in monitoring disease activity and the course of rheumatoid arthritis, but is not specific.

79
Q

An 18-year-old female basketball player comes to your office the day after sustaining an inversion injury to her ankle. She says she treated the injury overnight with rest, ice, compression, and elevation. You examine her and diagnose a moderate to severe lateral ankle sprain. In addition to rehabilitative exercises, you advise (check one)
a short-term cast
a posterior splint that allows no flexion or extension
a semi-rigid stirrup brace (Air-Stirrup, “Aircast”)
an elastic bandage
no external brace or support

A

a semi-rigid stirrup brace (Air-Stirrup, “Aircast”)

In acute ankle sprains, functional treatment with a semi-rigid brace that allows flexion and extension, or a soft lace-up brace is recommended over immobilization. Casting or posterior splinting is no longer recommended. Elastic bandaging does not offer the same lateral and medial support. External ankle support has been shown to improve proprioception.

80
Q

A 32-year-old female who is an avid runner presents with knee pain. You suspect patellofemoral pain syndrome. Which one of the following signs or symptoms would prompt an evaluation for an alternative diagnosis? (check one)
Peripatellar pain while running
Knee stiffness with sitting
A popping sensation in the knee
Locking of the joint
A positive J sign (lateral tracking of the patella when moved from flexion to full extension)

A

Locking of the joint

Patellofemoral pain syndrome is a clinical diagnosis and is the most common cause of knee pain in the outpatient setting. It is characterized by anterior knee pain, particularly with activities that overload the joint, such as stair climbing, running, and squatting. Patients complain of popping, catching, stiffness, and giving way. On examination there will be a positive J sign, with the patella moving from a medial to a lateral location when the knee is fully extended from the 90° position. This is caused by an imbalance in the medial and lateral forces acting on the patella. Locking is not characteristic of patellofemoral pain syndrome, so a loose body or meniscal tear should be considered if this is reported.

81
Q

A 45-year-old female presents with pain in her right hand and wrist. She is right hand–dominant and most symptoms are in the right hand and wrist. She has been awakening at night with numbness of the hand that improves after she shakes her wrist. Her work as a software engineer involves lots of typing.

On examination you note a positive Tinel’s sign at the right wrist consistent with carpal tunnel syndrome. She has an important project at work over the next several months and will not be able to take any time off.

Which one of the following treatments has the best evidence for delaying the need for definitive surgical therapy? (check one)
Night splints
Physical therapy
Therapeutic ultrasound
Oral corticosteroids
Corticosteroid injection

A

Corticosteroid injection

Carpal tunnel syndrome of mild to moderate severity can be treated nonsurgically. Patients with severe
symptoms or nerve damage seen on electromyography should be referred for surgical therapy. Nonsurgical
management options include splinting, physical therapy, therapeutic ultrasound, and corticosteroids (oral
or injection). Oral prednisone, 20 mg daily, for 10–14 days improves symptoms and function compared
with placebo, for up to 8 weeks, but oral corticosteroids are less effective than corticosteroid injections.
In a 2013 double-blind, placebo-controlled, randomized clinical trial comparing methylprednisolone
injection (40 mg and 80 mg) to saline injection, patients in the 80-mg injection group were less likely to
have surgery at 12 months.
While corticosteroid injections have the best evidence for delaying the need for surgery, night splints,
physical therapy, and therapeutic ultrasound have some evidence of benefit. Splinting was found to be
effective in a Cochrane review. It is low cost and safe and especially recommended in pregnancy-related
disease. Limited evidence supports the use of physical therapy to treat carpal tunnel syndrome. Nerve glide
exercises are simple hand and finger movements that are easy to learn, can be performed at home, and can
be combined with other treatments such as splinting. Therapeutic ultrasound also has limited evidence of
benefitIt requires an experienced therapist and requires multiple sessions, typically 5 days/week for 2–4
weeks.

82
Q

A 20-year-old college football player becomes disoriented and weak during an afternoon practice in the heat. On the field, he is sweaty and tachycardic, with a core (rectal) temperature of 40.9°C (105.6°F).

In this patient with exertional heatstroke, which one of the following would be the most important initial step to reduce his core body temperature? (check one)
Administering acetaminophen
Administering aspirin
Immersing him in cold water
Initiating intravenous fluids
Transporting him to an emergency facility for treatment

A

Immersing him in cold water

Urgent alleviation of hyperthermia is the primary objective in the treatment of heatstroke. Treatment should be started in the field with immersion in cold water or, if this is not an option, pouring copious amounts of water on the patient and fanning. No pharmacologic agents are indicated for temperature reduction in this situation, and acetaminophen and aspirin may exacerbate coagulopathy and liver injury in this situation. Infusion of cold fluids would be one of the treatments of choice in elderly patients with classic heatstroke. Transfer to an emergency facility would not be the most important initial step for this patient since treatment should be started on the field.

83
Q

An 18-year-old football player collapses on the field at the beginning of summer conditioning workouts. There was no obvious contact or injury. Upon assessment, he is awake but somnolent and diaphoretic. He reports a headache and is unable to identify where he is or the day of the week. His core temperature is 40.2°C (104.4°F).

Which one of the following is the most likely diagnosis? (check one)
Exercise-associated collapse
Heat edema
Heat exhaustion
Heat injury
Heatstroke

A

Heatstroke

The prompt recognition of heatstroke is critical to effective treatment. Heatstroke is characterized by a core temperature >40°C (104°F) in association with neurologic abnormalities such as headache, confusion, altered mental status, irritability, and seizure. Exercise-associated collapse, previously called heat syncope, generally occurs immediately after strenuous exercise and is more associated with hydration status. Heat edema is a benign condition manifested by mild swelling in the extremities and facial flushing in a patient with a normal temperature. Heat exhaustion may involve neurologic symptoms but is associated with a lower temperature (38.3°C–40.0°C [101°F–104°F]) and thus a better outcome. Like heatstroke, heat injury can be associated with a temperature >40°C, but does not involve neurologic symptoms. Instead, kidney, muscle, or liver injury may be present.

84
Q

A 19-year-old college freshman consults you at the request of her cross-country coach because she has not had a period in 2 of the last 3 months. She notes that her current training regimen is much more intense than in high school last year. She has an appropriate body image and denies caloric restriction. A pregnancy test at the student health center was negative. On examination she is lean and highly trained. Her examination is otherwise normal.
Which one of the following would be the most appropriate recommendation for this patient?
(check one)
Estrogen supplementation
Cyclic oral contraceptive pills
Increased caloric intake
Bisphosphonate therapy
Discontinuation of elite-level athletics

A

Increased caloric intake

This patient has exercise-related oligomenorrhea, but does not have the eating disorder that characterizes the female athlete triad. Menstrual problems in athletes do correlate with bone density loss and impaired recovery from exercise. Additionally, menstrual irregularity of varying severity is extremely common in female distance runners, perhaps affecting as many as 60%. Hormonal manipulation has not been shown to affect bone density, though it may produce withdrawal bleeding. Bisphosphonate therapy has been shown to be ineffective, and is not recommended in women of child-bearing age.
The main issue in well-nourished female athletes seems to be that energy intake is not increased to match energy expenditures at high levels of training. Unlike those with the female athlete triad, there is little evidence that athletes without eating disorders suffer substantial harm from exercise-induced menstrual problems. Ending an athletic career for this reason alone is not justified.